Del operator in spherical coordinates

Click For Summary
The del operator in spherical coordinates is derived by transforming the Cartesian coordinates into spherical coordinates using the chain rule. The spherical unit vectors are defined, with \hat{r}, \hat{\phi}, and \hat{\theta} expressed in terms of Cartesian unit vectors. To find the del operator, one must express the partial derivatives in Cartesian coordinates in terms of spherical coordinates and then simplify the expression. This process can be complex and may require consulting additional resources for clarity. Understanding these transformations is essential for applying the del operator in spherical coordinates effectively.
tirwit
Messages
16
Reaction score
0

Homework Statement


Write the del operator in spherical coordinates?


Homework Equations


I wrote the spherical unit vectors:
\hat{r}=sin\theta.cos\phi.\hat{x}+sin\theta.sin\phi.\hat{y}+cos\theta.\hat{z}
\hat{\phi}=-sin\phi.\hat{x}+cos\phi.\hat{y}
\hat{\theta}=cos\phi.cos\theta.\hat{x}+sin\phi+cos\theta.\hat{y}-sin\theta.\hat{z}


The Attempt at a Solution


I have no idea where to start... Please help, I'm going crazy with this...
 
Physics news on Phys.org
In cartesian coordinates, you have

\nabla = \hat{x} \frac{\partial}{\partial x}+\hat{y} \frac{\partial}{\partial y}+\hat{z} \frac{\partial}{\partial z}

Use the chain rule to change variables to r, \varphi, \vartheta. For example, you can write

\frac{\partial}{\partial x} = \frac{\partial r}{\partial x}\frac{\partial}{\partial r}+\frac{\partial \varphi}{\partial x}\frac{\partial}{\partial \varphi}+\frac{\partial \vartheta}{\partial x}\frac{\partial}{\partial \vartheta}

Then express the cartesian unit vectors in terms of the spherical unit vectors. Plug everything in and simplify.
 
It seems I suggested one of the more painful ways of finding the gradient.

Here's a web page that describes several different methods you can try:

http://math.mit.edu/classes/18.013A/HTML/chapter09/section04.html
 
Question: A clock's minute hand has length 4 and its hour hand has length 3. What is the distance between the tips at the moment when it is increasing most rapidly?(Putnam Exam Question) Answer: Making assumption that both the hands moves at constant angular velocities, the answer is ## \sqrt{7} .## But don't you think this assumption is somewhat doubtful and wrong?

Similar threads

  • · Replies 14 ·
Replies
14
Views
2K
  • · Replies 5 ·
Replies
5
Views
2K
Replies
3
Views
2K
Replies
33
Views
4K
  • · Replies 7 ·
Replies
7
Views
3K
Replies
2
Views
1K
  • · Replies 12 ·
Replies
12
Views
2K
  • · Replies 7 ·
Replies
7
Views
2K
  • · Replies 1 ·
Replies
1
Views
2K
  • · Replies 5 ·
Replies
5
Views
3K